Answer: -1(5a-2)(3a+1)
Step-by-step explanation:
Answer:
-(5a-2)(3a+1)
Step-by-step explanation:
he equation y equals 4 times 6 to the power of t shows the number of infected people from an outbreak of the measles. The variable y represents the number of infected people, and t represents time in weeks.
In how many weeks will the number of infected people reach 568?
The number of infected people will reach 568 in approximately 4 weeks
Linear equation and functionExponential equations are inverse of logarithmic equation and are represented by the equation below:
y = ab^x
where
a is the base value and x is the exponent
Given the equation that shows the number of infected people from an outbreak of the measles as:
y = 4(6)^t
If the number of infected people reach 568, hence;
568 = 4(6)^t
568/4 = 4(6)^t/4
6^t = 568/4
6^t = 142
Take the natural logarithm of both sides
ln(6^t) = ln142
t = ln142/ln6
t = 4.37
Hence the number of infected people will reach 568 in approximately 4 weeks
Learn more on exponential equation here: https://brainly.com/question/12940982
#SPJ1
if a + b = 23 and a= 47 what would B be?
Answer:
B = -24
Step-by-step explanation:
a + b = 23
Since a = 47
47 + b = 23
Subtract 47 from each side
47 - 47 + b = 23 - 47
Therefore:
B = -24
Answer:
b = -24
Step-by-step explanation:
a + b = 23
Substitute 47 for a.
47 + b = 23
Rearrange the equation.
23 - 47 = b
-24 = b
Check your answer by substituting the value you found for b into the equation.
47 + (-24) = 23
Find the mode for the following distribution. Number Frequency 36 3 40 5 44 9 48 7 52 7 56 5 60 3 What is the mode? 48 44 52 40
The mode for the following distribution is-
Number Frequency 36, 3, 40, 5 ,44, 9, 48, 7 ,52 7, 56, 5, 60, 3
is 56.
Mode is most occurring number is 56.
One of the values of the measurements of central tendency is the mode. This figure offers us an approximate notion of which elements in a data collection appear the most frequently.
For example, you may be aware that a college offers ten distinct courses to students. Now, out of them, the course with the most student registrations will be counted as the mode of our supplied data (number of students taking each course). Overall, mode notifies us about the item with the highest frequency in the data set.
The mode is one of the values of the central tendency measurements. This number offers us a sense of which elements in a data collection appear the most frequently.
For example, you may be aware that a college offers ten distinct courses to students. Out of these, the course with the most student registrations will be counted as the mode of our supplied data (number of students taking each course). Overall, mode informs us about the most frequent item in the data collection.
To learn more about Mode from the given link
https://brainly.com/question/17727138
#SPJ9
Answer:
The answer is 44, as it occurs most frequently. The easiest way to remember this is "mode" equals "most".
The sum of the squares of three consecutive odd integers is 83. Find the middle odd integer
Let [tex]2n+1[/tex] be the smallest of these integers, so the next two are [tex]2n+3[/tex] and [tex]2n+5[/tex]. The sum of their squares is 83, so
[tex](2n+1)^2 + (2n+3)^2 + (2n+4)^2 = 83[/tex]
Let [tex]x=2n+3[/tex] be the middle number. By substitution, we rewrite the equation as
[tex](x-2)^2 + x^2 + (x+2)^2 = 83[/tex]
Solve for [tex]x[/tex]. Expand the left side.
[tex](x^2 - 4x + 4) + x^2 + (x^2 + 4x + 4) = 83[/tex]
[tex]3x^2 + 8 = 83[/tex]
[tex]3x^2 = 75[/tex]
[tex]x^2 = 25[/tex]
[tex]\implies \boxed{x=\pm5}[/tex]
In a given bag of M & M's, 16 were yellow, 15 were green, 21 were brown, and 11 were red.
How many M&M's were in the bag?
What fraction were yellow?
What fraction were green?
What fraction were brown?
What fraction were red?
Answer:
There are 63 M&M's in the bag when all of them are added together.
16 out of 63 (16/63) were yellow.
5 out of 21 (5/21) were green. This answer is simplified. The original is 15/63.
1 out of 3 (1/3) were brown. This answer is simplified. The original is 21/63.
11 out of 63 (11/63) were red.
Tip:
To find the fraction of something, put the total on the bottom and the wanted portion on the top.
Example: 5 yellow and 10 red birds were in a cage. Which fraction were yellow?
We put the total of birds (5+10 = 15) on the bottom and the portion wanted ("5 yellow") on the top. The fraction is 5 out of 15 (5/15). This fraction can be simplified because 5 goes into 15 three times (5 x 3 = 15), so divide the fraction 5/15 by 3 to get 1/3. (5 goes into 5 one time.)
verbal expression for 2n + 7
Answer
Two times a number plus seven
Step-by-step explanation:
Hope this helps!!!
EXAMPLE 7 Solve: (a) 2t=80 (b) -6.02-8.6t
Strategy
Why
We will use a property of equality to isolate the variable on one side of the
equation.
Need Help?
To solve the original equation, we want to find a simpler equivalent equation of the
form t= a number or a number = t, whose solution is obvious.
Solution (a) To isolate t on the left-hand side, we can undo the multiplication by 2 by dividing both sides by 2.
2t = 80
This is the equation to solve.
Use the division property of equality: Divide both sides by 2.
2t
2
Self Check 7 Solve:
=
Read It
Simplify (21)/2 by removing the common factor of 2 in the numerator and denominator: 2/2 = 1.
The product of 1 and any number is that number: 1t = t.
t = 40
If we substitute 40 for t in 2t = 80, we obtain the true statement 80 = 80. This verifies that 40 is the solution. The solution set is (40).
1t=
(b) To isolate t on the right side, we use the division property of equality. We can undo the multiplication -8.6 by dividing both sides by -8.6.
This is the equation to solve.
-6.02-8.6t
Use the division property of equality: Divide both sides by -8.6.
Do the division: 8.6)6.02. The quotient of two negative numbers is positive.
80
-6.02 -8.6t
-8.6
The solution is 0.7. Verify that this is correct by checking.
W
(a) 18x=234
X=
Watch It
(b) 15.66= -0.6r
r=
It should be noted that the value of t in the equation 2t = 80, t will be 40.
How to illustrate the information?It should be noted that an equation is simony used to show the relationship between the variables or numbers that are given.
Therefore, it should be noted that solving the equation will be:
2t = 80
Divide through by 2
2t / 2 = 80 / 2
t = 40
Therefore, the value of t is 40.
Therefore, it should be noted that the value of t on the equation 2t = 80, t will be 40.
Learn more about equations on:
brainly.com/question/13763238
#SPJ1
Two parallel lines cut by a transversal. The transversal makes 4 angles with the top line. Angles 1 and 2 are above the line and angles 3 and 4 are between the lines. The transversal also makes 4 angles with the bottom line. Angles 5 and 6 are between the lines and angles 7 and 8 are below the bottom line. Between the lines, angles 3 and 6 are on opposite sides of the transversal and angles 4 and 5 are on opposite sides of the transversal. A. ∠3 ≅ ∠2 because they are alternate interior angles. B. m∠1 + m∠3 = 180 because they form a straight angle. C. ∠3 ≅ ∠6 because they are alternate interior angles. D. ∠1 and ∠6 are supplementary because ∠3 ≅ ∠6 and m∠1 + m∠3 = 180. E. ∠1 ≅ ∠3 because they are vertical angles.
Based on this parallel lines cut by a transversal, the true statements include the following:
B. m∠1 + m∠3 = 180 because they form a straight angle.
C. ∠3 ≅ ∠6 because they are alternate interior angles.
D. ∠1 and ∠6 are supplementary because ∠3 ≅ ∠6 and m∠1 + m∠3 = 180.
What are parallel lines?Parallel lines can be defined as two (2) lines that are always the same (equal) distance apart and never meet.
What is the Alternate Interior Angles Theorem?The Alternate Interior Angles Theorem states that when two (2) parallel lines are cut through by a transversal, the alternate interior angles that are formed are congruent.
What is a supplementary angle?A supplementary angle can be defined as two (2) angles or arc whose sum is equal to 180 degrees (180°).
In conclusion, ∠1 and ∠6 are supplementary angles because ∠3 is congruent to ∠6 and m∠1 + m∠3 = 180 degrees as shown in the image attached below.
Read more on parallel lines here: brainly.com/question/2497532
#SPJ1
Complete Question:
Select all the true statements. Two parallel lines cut by a transversal. The transversal makes 4 angles with the top line. Angles 1 and 2 are above the line and angles 3 and 4 are between the lines. The transversal also makes 4 angles with the bottom line. Angles 5 and 6 are between the lines and angles 7 and 8 are below the bottom line. Between the lines, angles 3 and 6 are on opposite sides of the transversal and angles 4 and 5 are on opposite sides of the transversal.
A. ∠3 ≅ ∠2 because they are alternate interior angles.
B. m∠1 + m∠3 = 180 because they form a straight angle.
C. ∠3 ≅ ∠6 because they are alternate interior angles.
D. ∠1 and ∠6 are supplementary because ∠3 ≅ ∠6 and m∠1 + m∠3 = 180.
E. ∠1 ≅ ∠3 because they are vertical angles.
Find the slope of the line graphed below.
Answer:
m=4/3
Step-by-step explanation:
slope=rise/run or y=mx+b
rise/run=8/6=4/3
3
For the equation, y=x-1, determine the slope of the line (a) parallel to the given line and (b) perpendicular to the given line.
(a) Select the correct choice below and, if necessary, fill in the answer box to complete your choice.
OA. The slope of the line parallel to the given line is
(Type an integer or a fraction.)
OB. The slope of the line parallel to the given line is not defined.
(b) Select the correct choice below and, if necessary, fill in the answer box to complete your choice.
O A. The slope of the line perpendicular to the given line is
(Type an integer or a fraction.)
...
OB. The slope of the line perpendicular to the given line is not defined.
h
This question: point(s) P
a. The slope of the line parallel to y = 3/4x - 1 is: 3/4.
b. The slope of the line perpendicular to y = 3/4x - 1 is: -4/3.
What is the Slope of Lines that are Perpendicular and Lines that are Parallel?If two lines are parallel to each other, they will have the same slope value. On the other hand, if the two lines are perpendicular to each other, they will have slopes that are negative reciprocals of each other.
Thus, given the equation, y = 3/4x - 1, the slope of this line is 3/4.
The negative reciprocal of 3/4 is -4/3.
Therefore, we can conclude that:
a. The slope of the line parallel to y = 3/4x - 1 is: 3/4.
b. The slope of the line perpendicular to y = 3/4x - 1 is: -4/3.
Learn more about the slope of parallel and perpendicular lines on:
https://brainly.com/question/13657035
#SPJ1
a kind of sampling strategy least likely to produce statistics that are good estimates of population perameters is a
A kind of sampling strategy least likely to produce statistics that are good estimates of population parameters is a haphazard sample.
What is haphazard sampling ?A haphazard sampling is method of sampling in which the performer does not intend to use a systematic approach for choosing a sample rather they do the random selection without using anu conscious bias.
Haphazard sampling assumes that all population units are alike but unless they are really similar there is always a chance of selection falling prey to biases. Therefore, haphazard sample is least likely to produce statistics that are good estimates of population parameters.
Also, Learn more about haphazard sample from:
https://brainly.com/question/18958253
#SPJ1
You are building a shelf that fits in a corner in the figure the entire shelf is EGH. each unit in the coordinate plane represents one inch. Find the area A of the shelf. E is (25,20) G is (15,10) H is (25,10)
Answer:
Step-by-step explanation:
Complete Question: You are building a shelf that fits in a corner in the figure the entire shelf is XYZ each unit in the coordinate plane represents one inch find the area of the shelf.
Answer:
112.5 units²
Step-by-step explanation:
Area of the shelf = ½*base*height.
The base is the distance between Z(0, 5) and Y(15, 5)
The height is the distance between X(15, 20) and Y(15, 5).
Therefore area of the shelf = ½*ZY*XY
Use the distance formula to find ZY and XY.
Distance between Z(0, 5) and Y(15, 5):
Let,
Distance between X(15, 20) and Y(15, 5):
Let,
Area of shelf = ½*15*15 = ½*225 = 112.5 units²
Show that the set {-1, 0, 1} is closed under multiplication
The statement that the set {-1, 0, 1} is closed under multiplication, is true and can be proven.
As per the question statement, we are provided with a set {-1, 0, 1}.
We are required to determine if the above mentioned set is closed under multiplication or not.
To solve this question, we simply need to verify if the products of every distinct combination of 2 elements from the concerned set, belongs to the set itself or not, i.e., if we take any two elements "x" and "y", say, from the set "Z", then, [(x*y) ∈ Z].
Here, [tex][(-1)*0]=0[/tex], i.e., belongs to set {-1, 0, 1}.
[tex][(-1)*1]=(-1)[/tex] i.e., also belongs to set {-1, 0, 1}.
Finally, [tex](0*1)=0[/tex] i.e., also belongs to set {-1, 0, 1}.
Since, the products of every distinct combination of 2 elements from the concerned set, belongs to the set itself, the statement that the set
{-1, 0, 1} is closed under multiplication, is true and proven.
Set closed under multiplication: This means that, for any set say "Z", the product of distinct combinations of it's elements, say (x*y) belongs to Z, such that [(x, y) ∈ Z].To learn more about Sets closed under multiplication, click on the link below.
https://brainly.com/question/15088185
#SPJ1
x−5≥−8 Solve the inequality for x. Graph the solution.
help me in economics please see the picture
Answer:
12
Step-by-step explanation:
Given a table of supply and demand at each price point, you are asked for the equilibrium price if the demand numbers are decreased by 0.5 million.
Equilibrium priceThe equilibrium price is the price at which the quantity demanded is equal to the quantity supplied.
For the given table, we see that a price of $14 thousands will result in both demand and supply being 2.00 millions per year. That is the equilibrium price before any adjustment to the table is made.
The problem statement is telling you to use a table that has all of the demand values decreased by 0.5 million. That makes the demand column read ...
2.25, 2.00, 1.75, 1.50, 1.25, 1.00, 0.75
This new column of reduced demand values matches the numbers in the supply column when they are both 1.75 million per year. That match occurs on the line with price $12 thousand.
The new equilibrium price is $12 thousand.
Which of the following represents
this graph?
-1
a. |2x - 3|< 4
b.x - 3|< 12
c. |2y - 7 > 9
d. 5x + 2 <3
8
Answer:
c
Step-by-step explanation:
The average of -1 and 8 is 7/2, so the absolute value should equal 0 at 7/2.
The only option that satisfies this is c.
HL - Segment Proofs
6 of 86 of 8 Items
Question
Write the letter of the property, definition, or postulate that justifies each statement.
If 2XY = YZ, then YZ = 12YZ
Addition Property of Equality
Addition Property of Equality
Subtraction Property of Equality
Subtraction Property of Equality
Multiplication Property of Equality
Multiplication Property of Equality
Division Property of Equality
Division Property of Equality
Substitution Property
Substitution Property
Reflexive Property
Reflexive Property
Symmetric Property
Symmetric Property
Transitive Property
Transitive Property
Definition of Congruence
Definition of Congruence
Definition of Midpoint
Definition of Midpoint
Segment Addition Postulate
If 2XY = YZ then XY = 1/2YZ this is division property of equality.
What are some properties of equality ?Division property of equality, multiplication property of equality, subtraction property of equality and addition property of equality.
According to the given question we have to define each property that we used to solve or interpret if 2XY = YZ, then XY = 1/2YZ.
This is division property of equality.
2XY = YZ
Now if we divide both sides of the equality by 2 we'll get
XY = 1/2 YZ and we know diving by a same constant on both sides of the inequality doesn't change the equality relation.
If 2XY = YZ, then YZ = 12YZ is not the correct question,The correct question is If 2XY = YZ then XY = 1/2YZ.
learn more about properties of equality here :
https://brainly.com/question/15721307
#SPJ1
The steps to solve the equation 7(x + 3) = 9x - 4x − 35 are listed below. Put the justifications for the steps in the correct order to correspond with the steps.
7(x + 3) = 9x - 4x + 37
Step 1: 7x + 21 = 9x - 4x + 37
Step 2: 7x+ 21 - 21 = 9x - 4x +37 - 21
Step 3: 7x = 5x +16
Step 4: 7x - 5x = 5x - 5x +16
Step 5: 2x = 16
Step 6: 2x ÷ 2 = 16 ÷ 2
Step 7: x = 54
-Reorder Answers-
Subtraction Property of Equality
Simplification
Distributive Property
Subtraction Property of Equality
Combine like terms
Combine like terms
Division Property of Equality
The answer include the following:
Step 1: 7x + 21 = 9x - 4x + 37 - distributive property.
Step 2: 7x+ 21 - 21 = 9x - 4x +37 - 21 - subtraction property
Step 3: 7x = 5x +16 - combine like terms.
Step 4: 7x - 5x = 5x - 5x +16 - subtraction property.
Step 5: 2x = 16 - combine like terms
Step 6: 2x ÷ 2 = 16 ÷ 2 - division property.
Step 7: x = 54 - simplification.
What is Simplification?This is referred to as process in which a mathematical expression is replaced by another so as to ensure that it is simpler and shorter in other solve the problem and for better understanding.
In the case of 7(x + 3) = 9x - 4x − 35, distributive property is used to expand the bracket and collecting of like terms are also used for easier subtraction and addition process
This employs the use of different arithmetic operations such as division, etc and the steps above shoes the most appropriate justifications in this type of scenario.
Read more about Equation here https://brainly.com/question/18831322
#SPJ1
How much interest would 2000 earn, with simple interest, in two years at the rate of 1.2%?
The simple interest, in two years at the rate of 1.2% is 880
In this case, we must calculate how much interest 2000 would earn compounded yearly over two years at a rate of 1.2.
I = 880 in interest earned
Annually, the system may be evaluated in terms of simple interest.
A = P(rt) for the first year, where r = 1.2 and t = 1.
Therefore;
A = 2000(1.2×1).
A = 2400.
As a result, the revised Principal amount for the second year is 2400.
As a result, A' = 2400 (1.21).
A' = 2880.
As a result, the sum owing after two years is 2880.
However, because the initial principal is $2,000,
The interest is defined as I = A' - P, or I = 2880 - 2000.
I = 880 as a result of interest.
Simple interest is computed as S.I. = P R T, where P = Principal, R = Annual Rate of Interest in %, and T = Time, commonly expressed as the number of years. The interest rate is expressed as a percentage r% and is written as r/100.
The principle is the money borrowed from the bank or invested at the outset. P represents the principal.
Rate: The rate of interest at which the principle amount is handed to someone for a specific period of time; the rate of interest can be 5%, 10%, or 13%, for example. R represents the interest rate.
Time is the length of time for which the primary amount is supplied to someone. T represents time.
To learn more about Simple interest from the given link:
https://brainly.com/question/25845758
#SPJ9
Emma has a points card for a movie theater.
She receives 30 rewards points just for signing up.
She earns 7.5 points for each visit to the movie theater.
She needs at least 135 points for a free movie ticket.
Write and solve an inequality which can be used to determine vv, the number of visits Emma can make to earn her first free movie ticket.
Inequality:
v:
The inequality is 30 + 7.5v ≤ 135
The number of visits Emma could make to earn her first free movie ticket is 14
Writing an InequalityFrom the question, we are to write and solve an inequality which can be used to determine the number of visits Emma can make to earn her first fee movie ticket
From the given information,
"She receives 30 rewards points just for signing up"
and
"She earns 7.5 points for each visit to the movie theater"
If she visits the movie theater v number of times,
Then,
She would earn
30 + 7.5v points
Also,
She needs at least 135 points for a free movie ticket.
Thus,
The inequality that could be used to determine the number of visits, v, to earn a free movies ticket is
30 + 7.5v ≤ 135
Solving the inequality
30 + 7.5v ≤ 135
Subtract 30 from both sides
30 - 30 + 7.5v ≤ 135 - 30
7.5v ≤ 105
v ≤ 105/7.5
v ≤ 14
Hence, the number of visits Emma could make to earn her first free movie ticket is 14
Learn more on Writing an inequality here: https://brainly.com/question/22558824
#SPJ1
LESSON 7.3 HELPPPPPPPPPPPPPPPP!!!!!!!!!!!!!!!!!!!!!!!!!!!!!!!
THERE ARE 4 DIFFERENT QUESTIONS PLEASE ANSWER ALL 4 TO GET THE POINTS THANKS YOU
Find the missing side lengths. Leave your answers as radicals in simplest form.
Using relations in a right triangle, the missing side lengths are given as follows:
1. D. [tex]x = 7\sqrt{2}, y = 7[/tex]
2. B. [tex]x = 3\sqrt{2}, y = 2[/tex].
3. C. x = 6, y = 3.
4. C. [tex]x = 2\sqrt{5}, y = \frac{2\sqrt{15}}{3}[/tex]
What are the relations in a right triangle?There are three relations in a right triangle, given as follows:
The sine of an angle of the triangle is given by the length of the opposite side to the angle divided by the length of the hypotenuse.The cosine of an angle of the triangle is given by the length of the adjacent side to the angle divided by the length of the hypotenuse.The tangent of an angle of the triangle is given by the length of the opposite side to the angle divided by the length of the adjacent side to the angle.For the first triangle, we have that:
tan(45º) = 7/y
1 = 7/y
y = 7.
Applying the Pythagorean Theorem:
x² = 7² + 7²
x = sqrt(2 x 49)
[tex]x = 7\sqrt{2}[/tex]
Hence the correct option is:
D. [tex]x = 7\sqrt{2}, y = 7[/tex]
For the second triangle, we have that:
tan(45º) = y/3
1 = y/3
y = 3.
Applying the Pythagorean Theorem:
x² = 3² + 3²
x = sqrt(2 x 9)
[tex]x = 3\sqrt{2}[/tex]
Hence the correct option is:
B. [tex]x = 3\sqrt{2}, y = 2[/tex].
For the third triangle, we have that:
[tex]\tan{30^\circ} = \frac{y}{3\sqrt{3}}[/tex]
[tex]\frac{\sqrt{3}}{3} = \frac{y}{3\sqrt{3}}[/tex]
y = 3 x 3/3
y = 3.
Then:
[tex]\sin{30^\circ} = \frac{y}{x}[/tex]
0.5 = 3/x
x = 6.
Hence the correct option is:
C. x = 6, y = 3.
For the fourth triangle, we have that:
[tex]\sin{30^\circ} = \frac{y}{\frac{4\sqrt{15}}{3}}[/tex]
[tex]0.5 = \frac{y}{\frac{4\sqrt{15}}{3}}[/tex]
[tex]y = \frac{2\sqrt{15}}{3}[/tex]
[tex]\cos{30^\circ} = \frac{x}{\frac{4\sqrt{15}}{3}}[/tex]
[tex]\frac{\sqrt{3}}{{2}} = \frac{x}{\frac{4\sqrt{15}}{3}}[/tex]
[tex]x = 2\sqrt{5}[/tex]
Hence the correct option is:
C. [tex]x = 2\sqrt{5}, y = \frac{2\sqrt{15}}{3}[/tex]
More can be learned about relations in a right triangle at https://brainly.com/question/26396675
#SPJ1
What is the value of x?
Enter your answer in the box.
The value of x in the given object is 12.
What is the value of x?The given shape is an equilateral triangle. A triangle is a three-sided polygon with three edges and vertices. The sum of angles in a triangle is equal to 180 degrees.
An equilateral triangle is a triangle in which the three angles are equal. This means that each angle in an equilateral triangle measures 60 degrees. Also, in an equilateral triangle the three sides are equal.
Since, all the three sides of an equilateral triangle is equal, the expressions for their sides can be equated to each other to determine the value of x
4x - 10 = 5x - 22
Combine similar terms: 22 - 10 = 5x - 4x
Add similar terms: 12 = x
To learn more about triangles, please check: https://brainly.com/question/22949981
#SPJ1
Trig question for a test prep assignment ungraded before the final
According to the solving the angular speed of the wheel in radius per minutes would be:
100.
The correct option is 3.
What is angular velocity?angular velocity is the rate at which an item rotates or revolves around an axis, or the pace that the angular displacement among two bodies varies. This displacement is shown in the diagram by the angle formed by a line with one body and a line on the opposite body.
What is the formula for calculating angular velocity?Angular velocity is defined as the rate of change of an object's position angle with respect to time, therefore w = theta / t, where w denotes angular velocity, theta denotes position angle, and t denotes time.
What are the definitions of angular instantaneous velocity?The angular speed of a rotating object is a scalar measurement. The angular velocity of a rotating object is measured as a vector. Only the magnitude of angular speed is specified. The magnitude and direction of angular velocity are specified.
According to the given:Radius of the wheel = 2ft
velocity of the wheel = 200 ft/min
We know that the formula of Angular velocity:
ω = ν/r
ω = output
ν = liner velocity
r = path radius
Substituting the value in the formula:
ω = ν/r
ω = 200/2
= 100
According to the solving the angular speed of the wheel in radius per minutes would be:
100.
To know more about Angular velocity visit:
https://brainly.com/question/12446100
#SPJ9
I’m confused pls help
Answer: answer is 3
Step-by-step explanation: it is 3 because you do 1 d.p. plus a and b and you get 3
PLS HELP THIS IS DUE FIRST THING TMR
Answer:
m<a=45
m<b=90
m<c=45
m<d=45
m<e=90
m<g=130
m<h=50
m<i=130
m<k=85
m<l=95
m<n=95
At the beginning of June Max and Sean start saving money for the fair which occurs 10 weeks later Sean starts out with $20 saves just $2 per week Max starts out with no money but says $5 per week after how many weeks will Max have more money than Shawn explain how you found your answer
Max is going to have more money than Shawn at the end of the seven weeks from the calculations
How to solve for the amount of moneyWe would first have to define x to be the point where the money that they have would be equal
This is the week that is referred to as x
Sean would have $20 + $2x
Max would have $5x
When both amounts are equal, we would have sean = max
= $20 + $2x = $5x
Take like terms
20 = 5x - 2x
20 = 3x
divide through by 3
x = 20 / 3
= 6.6667
Remember
In 7 weeks sean = 20 + 2x 7 = 34
max = 5 x 7 = 75
Hence we can say that Max would have more money than Sean
Read more on quantities here:
https://brainly.com/question/22375193?referrer=searchResults
#SPJ1
Yossi used to have a square garage with ft of floor space. recently built an addition to it. The garage is still a square, but now it has 50% more floor space. What was the length of one side of the garage originally? What is the length of one side of the garage now? What was the percent increase in the length of one side?
The length of one side of the garage originally is 18.17 feet.
The length of one side of the garage now is 22.25 feet.
The percent increase is 22.4%.
What is the increase in the length of the garage?A square is a quadrilateral with four sides of equal length. A square has two diagonals of equal length that intersect at right angles.
Given the area of a square, the length of one side can be determined by calculating the square root of the area.
Area of the square = length²
Length of one side of the square = √area
√330 = 18.17 ft
Area of the garage after it was increased : (1 + percentage increase) x initial area
(1 + 0.5) x 330
1.5 x 330 = 495 feet²
Length of the side of the garage after it is increased = √495 = 22.25 feet
The percentage increase = (22.25 / 18.17) - 1 = 0.224 = 22.4%
Here is the complete question:
Yossi used to have a square garage with 330ft of floor space. recently built an addition to it. The garage is still a square, but now it has 50% more floor space. What was the length of one side of the garage originally? What is the length of one side of the garage now? What was the percent increase in the length of one side?
To learn more about a square, please check: https://brainly.com/question/9030544
#SPJ1
Students are raising money by selling wristbands. They buy 500 wristbands for just $0.90 each. They want to make at least $250.How much money should they charge for each wristband?
The amount that the students should charge for the wristbands will be at least $1.40.
How to illustrate the information?From the information, the students want to buy 500 wristbands for just $0.90 each and they want to make at least $250.
Therefore, the amount that they should charge for each will be:
Total cost = 500 × $0.90 = $450
Therefore, the amount charged be:
500x - 450 >= 250
Collect like terms
500x >= 250 + 450
500x >= 700
x >= 700/500
x>= $1.4
They should charge at least $1.40.
Learn more about inequalities on:
brainly.com/question/11613554
#SPJ1
Hi! Can someone please explain?I will give brainliest :)
Answer:
$8.20
Step-by-step explanation:
2.35+2.35=4.7
4.7+1.5=6.2
6.2+2=8.2
find the solution set the this inequality 2x -3 > 2x-5/2
Answer:
0> 1/2
Step-by-step explanation: